Last visit was: 06 May 2024, 23:41 It is currently 06 May 2024, 23:41

Close
GMAT Club Daily Prep
Thank you for using the timer - this advanced tool can estimate your performance and suggest more practice questions. We have subscribed you to Daily Prep Questions via email.

Customized
for You

we will pick new questions that match your level based on your Timer History

Track
Your Progress

every week, we’ll send you an estimated GMAT score based on your performance

Practice
Pays

we will pick new questions that match your level based on your Timer History
Not interested in getting valuable practice questions and articles delivered to your email? No problem, unsubscribe here.
Close
Request Expert Reply
Confirm Cancel
User avatar
Senior Manager
Senior Manager
Joined: 03 Sep 2006
Posts: 446
Own Kudos [?]: 6790 [0]
Given Kudos: 33
Send PM
User avatar
Manager
Manager
Joined: 25 Feb 2010
Posts: 208
Own Kudos [?]: 320 [1]
Given Kudos: 10
Send PM
User avatar
Manager
Manager
Joined: 21 Jul 2009
Posts: 218
Own Kudos [?]: 440 [1]
Given Kudos: 22
Concentration: World Domination, Finance, Political Corporatization, Marketing, Strategy
Schools:LBS, INSEAD, IMD, ISB - Anything with just 1 yr program.
 Q47  V32
Send PM
avatar
Manager
Manager
Joined: 04 May 2009
Posts: 109
Own Kudos [?]: 109 [0]
Given Kudos: 10
Location: London
Schools:Haas (WL), Kellogg (matricultating), Stanford (R2, ding), Columbia (ding)
 Q36  V31 GMAT 2: 730  Q48  V42
WE 1: 3 years hotel industry sales and marketing France
WE 2: 3 years financial industry marketing UK
Send PM
Re: CR5 [#permalink]
BarneyStinson wrote:

I say B.

For "evaluate the argument" kind of questions, the trick is to answer the given options with a Yes/No answer. For eg, look at option A, Yes, the population will increase / No, the population will not increase. If neither of the answers disturb the conclusion that Centennial is planning to convert office space into hotel rooms, that option can be discarded.

B is right because, Yes, demand for office space will increase / No, demand will not increase, directly hits the plan.


I agree with Barney and just wanted to add that the important point not to miss here in the conclusion is that the company makes this plan in order to "maximize" profits. But what if they could maximize profits actually by using the space for it already is, without investing more?

The answer to question in answer B will help determine exactly that.
User avatar
Director
Director
Joined: 17 Feb 2010
Posts: 633
Own Kudos [?]: 3226 [0]
Given Kudos: 6
Send PM
Re: CR5 [#permalink]
i narrowed it to B and E.

ended up picking (E).
Senior Manager
Senior Manager
Joined: 02 Oct 2009
Posts: 310
Own Kudos [?]: 3803 [0]
Given Kudos: 412
GMAT 1: 530 Q47 V17
GMAT 2: 710 Q50 V36
WE:Business Development (Consulting)
Send PM
Re: CR5 [#permalink]
E is a tricky Trap.

BarneyStinson`s method of solving the best way to attack these type of questions it will help u save ur time too...
User avatar
Manager
Manager
Joined: 21 Jan 2010
Posts: 114
Own Kudos [?]: 327 [0]
Given Kudos: 38
Send PM
Re: CR5 [#permalink]
B. same reason as above.
User avatar
Senior Manager
Senior Manager
Joined: 03 Sep 2006
Posts: 446
Own Kudos [?]: 6790 [0]
Given Kudos: 33
Send PM
Re: CR5 [#permalink]
BarneyStinson wrote:
LM wrote:
Please explain....


I say B.

For "evaluate the argument" kind of questions, the trick is to answer the given options with a Yes/No answer. For eg, look at option A, Yes, the population will increase / No, the population will not increase. If neither of the answers disturb the conclusion that Centennial is planning to convert office space into hotel rooms, that option can be discarded.

B is right because, Yes, demand for office space will increase / No, demand will not increase, directly hits the plan.


Thanks for the hint and +1 to you
User avatar
Intern
Intern
Joined: 01 Dec 2008
Posts: 18
Own Kudos [?]: 3 [0]
Given Kudos: 2
Send PM
Re: CR5 [#permalink]
B is the only option that the Real Estate developers have to be considered about.

A - The argument does not consider the local population growth- This is irrelevant
B - If the demand for office spaces (consequently the price) of per Sq foot area of office space is > than per sq foot area of a hotel room then clearly the Real estate developers would have made a wrong choice if they convert office space to hotel rooms.
C - The Estate developers need not be concerned with other travel related services, because they are not providing this service, hence won't affect their balance sheet.
D - Another irrelevant option.
E - If we observe the stimulus carefully the question talks only about hotel rooms. The stimulus does not provide any information about a possibility of distinction between hotels and office spaces converted to hotels. We can safely eliminate this choice.
User avatar
Director
Director
Joined: 25 Aug 2007
Posts: 520
Own Kudos [?]: 5429 [0]
Given Kudos: 40
WE 1: 3.5 yrs IT
WE 2: 2.5 yrs Retail chain
Send PM
Re: CR5 [#permalink]
LM,

Please select the tag (Argument Evaluation) and source of question so that the question can be categorised correctly.
User avatar
Manager
Manager
Joined: 11 May 2010
Posts: 109
Own Kudos [?]: 549 [0]
Given Kudos: 11
Send PM
Re: CR5 [#permalink]
I would go for B

If in the future the demand of offices is high then the manager should keep the office building. If it is not then he should change them to hotel rooms.



Archived Topic
Hi there,
This topic has been closed and archived due to inactivity or violation of community quality standards. No more replies are possible here.
Where to now? Join ongoing discussions on thousands of quality questions in our Critical Reasoning (CR) Forum
Still interested in this question? Check out the "Best Topics" block above for a better discussion on this exact question, as well as several more related questions.
Thank you for understanding, and happy exploring!
GMAT Club Bot
Re: CR5 [#permalink]
Moderators:
GMAT Club Verbal Expert
6922 posts
GMAT Club Verbal Expert
238 posts

Powered by phpBB © phpBB Group | Emoji artwork provided by EmojiOne